Sols2011D1

download Sols2011D1

of 3

Transcript of Sols2011D1

  • 8/3/2019 Sols2011D1

    1/3

    The 4th Romanian Master of Mathematics Competition Solutions

    Day 1: Friday, February 25, 2011, Bucharest

    Problem 1. Prove that there exist two functions

    f, g: RR,

    such that f g is strictly decreasing, while g f is strictlyincreasing.

    (POLAND) ANDRZEJ KOMISARSKI & MARCIN KUCZMA

    Solution. Let

    A=

    kZ

    22k+1,22k

    22k,22k+1

    ;

    B=

    kZ

    22k,22k1

    22k1,22k

    .

    Thus A= 2B, B= 2A, A= A, B=B, AB= , and finallyAB {0} =R. Let us take

    f(x) =

    x for x A;x for x B;0 for x= 0.

    Take g(x) = 2f(x). Thus f(g(x)) = f(2f(x)) = 2x andg(f(x)) = 2f(f(x)) = 2x.

    Problem 2. Determine all positive integers n for whichthere existsa polynomial f(x) with real coefficients, with thefollowing properties:

    (1) for eachinteger k, the number f(k)isanintegerifandonly ifk is not divisible byn;

    (2) the degree offis less than n.

    (H UNGARY) GZ AKS

    Solution. We will show that such polynomial exists if andonly ifn= 1 or nis a power of a prime.

    We will use two known facts stated in Lemmata 1 and 2.

    LEMMA 1. If pa is a power of a prime and k is an integer,

    then (k1)(k2)...(k pa

    +1)(pa1)! is divisible byp if and only

    ifk is not divisible bypa.

    Proof. First suppose that pa | kand consider

    (k1)(k2) (k pa+1)(pa1)! =

    k1pa1

    k2pa2

    k pa+11

    .

    In every fraction on the right-hand side, p has the samemaximal exponent in the numerator as in the denominator.

    Therefore, the product (which is an integer) is not divisiblebyp.

    Now suppose that pak. We have

    (k1)(k2) (k pa+1)(pa1)! =

    pa

    k k(k1) (k p

    a+1)(pa)!

    .

    The lastfraction is an integer. In the fraction pa

    k,thedenom-

    inator k is not divisible bypa.

    LEMMA 2. If g(x) is a polynomial with degree less than nthen

    n=0

    (1)

    n

    g(x+n) = 0.

    Proof. Apply induction on n. For n= 1 then g(x) is a con-stant and

    1

    0

    g(x+1)

    1

    1

    g(x) = g(x+1) g(x) = 0.

    Now assume that n> 1 and the Lemma holds for n1. Leth(x) = g(x+1) g(x); the degree ofhis less than the degreeofg, so the induction hypothesis applies for g and n1:

    n1=0

    (1)

    n1

    h(x+ n1) = 0

    n1=0

    (1)

    n1

    g(x+n) g(x+n1)= 0

    n1

    0

    g(x+n)+

    n1=1

    (1)

    n11

    +

    n1

    g(x+n) (1)n1

    n1n1

    g(x) = 0

    n=0

    (1)

    n

    g(x+n) = 0.

    LEMMA 3. Ifn has at least two distinct prime divisors thenthe greatest common divisor ofn

    1

    ,n

    2

    , . . . ,

    nn1

    is 1.

    Proof. Suppose to the contrary that p is a common primedivisor of

    n1

    , . . . ,

    nn1

    . In particular, p| n1= n. Let abe the

    exponent ofp in the prime factorization ofn. Since nhas atleast two prime divisors, we have 1 < pa < n. Hence, npa1and

    npa

    are listed among

    n1

    , . . . ,

    nn1

    and thus p | npa and

    p | npa1

    . But then p divides

    npa

    npa1

    = n1pa1

    , which

    contradicts Lemma 1.

  • 8/3/2019 Sols2011D1

    2/3

    2

    Next we construct the polynomial f(x) when n= 1 or nisa power of a prime.

    For n= 1, f(x) = 12 is such a polynomial.Ifn = pawhere p is a prime and a is a positive integer

    then let

    f(x) = 1p

    x1

    pa

    1

    = 1

    p (x1)(x2) (x p

    a+1)(pa

    1)!

    .

    The degree of this polynomial is pa1 = n1.The number (k1)(k2)(kp

    a+1)(pa1)! is an integer for any inte-

    ger k, and, by Lemma 1, it is divisible byp if and only ifk isnot divisible bypa = n.

    Finally we prove that ifn has at least two prime divisorsthen no polynomial f(x) satisfies (1,2). Suppose that somepolynomial f(x) satisfies (1,2), andapply Lemma 2 for g= fand x= kwhere 1 k n1. We get that

    n

    kf(0) = 0n,=k

    (1)kn

    f(k+).Since f(k),.. . ,f(1) and f(1),...,f(n k) are all integers,

    we conclude thatn

    k

    f(0) is an integer for every 1 k n1.

    By dint of Lemma 3, the greatest common divisor ofn1

    ,n

    2

    , . . . ,

    nn1

    is 1. Hence, there will exist some integers

    u1, u2, . . . ,un1 for which u1n

    1

    + +un1 nn1= 1. Thenf(0) =

    n1k=1

    uk

    n

    k

    f(0) =

    n1k=1

    uk

    n

    k

    f(0)

    is a sum of integers. This contradicts the fact that f(0) is notan integer. So such polynomial f(x) does not exist.

    Alternative Solution. (I. Bogdanov) We claim the answeris n= p for some prime pand nonnegative.

    LEMMA. For every integers a1, . . . , an there exists an integer-valued polynomial P(x) of degree < n such that P(k) = akfor all 1 k n.

    Proof. Induction on n. Forthe base case n= 1onemaysetP(x) = a1. For the induction step, suppose that the polyno-mial P1(x) satisfies the desired property for all 1 k n1.Then set P(x) = P1(x)+ (anP1(n))

    x1n1

    ; since

    k1n1

    = 0 for

    1

    k

    n

    1 and n1n1

    = 1, the polynomial P(x) is a soughtone. Now, if for some n there exists some polynomial f(x)

    satisfying the problem conditions, one may choose someinteger-valued polynomial P(x) (of degree < n1) coincid-ing with f(x) at points 1,..., n 1. The difference f1(x) =f(x) P(x) also satisfies the problem conditions, thereforewe may restrict ourselves to the polynomials vanishing atpoints 1,..., n 1 that are, the polynomials of the formf(x) = cn1

    i=1 (x i) for some (surely rational) constant c.

    Let c= p/q be its irreducible form, and q=dj=1 pjj be theprime decomposition of the denominator.

    1. Assume that a desired polynomial f(x) exists. Sincef(0) is not an integer, we have q(1)n1(n1)! and hencepjj

    (1)n1(n1)! for some j. Hencen1

    i=1

    (pjj

    i)

    (

    1)n1(n

    1)!

    0 (mod pjj

    ),

    therefore f(pii

    ) is not integer, too. By the condition (i), thismeans that n | pi

    i, and hence n should be a power of a

    prime.

    2. Now let us construct a desired polynomial f(x) for anypower of a prime n= p. We claim that the polynomial

    f(x) = 1p

    x1n1

    = n

    px

    x

    n

    fits. Actually, consider some integer x. From the first repre-sentation, the denominator of the irreducible form of f(x)

    may be 1 or p only. Ifp

    x, then the prime decompositionof the fraction n/(px) contains p with a nonnegative expo-nent; hence f(x) is integer. On the other hand, ifn= p | x,then the numbers x1, x2,..., x(n1) contain the sameexponents of primes as the numbers n1,n2,...,1respec-tively; hence the number

    x1n1

    =n1

    i=1 (x i)n1i=1 (n i)

    is not divisible byp. Thus f(x) is not an integer.

    Problem 3. A triangle ABC is inscribed in a circle . Avariable line chosen parallel to BC meets segments AB,

    AC at points D, E respectively, and meets at points K, L(where D lies between K and E). Circle 1 is tangent to thesegments K Dand BD and also tangent to , while circle 2is tangent to the segments LE and C Eand also tangent to.Determine the locus, as varies, of the meeting point of thecommon inner tangents to 1 and 2.

    (RUSSIA) VASILYMOKIN & FEDOR IVLEV

    Solution. Let P be the meeting point of the common in-ner tangents to 1 and 2. Also, let b be the angle bisec-tor ofB AC. Since K L BC, b is also the angle bisectorofK AL.

    LetH

    be the composition of the symmetryS

    with respectto band the inversionI of centre Aand ratio AK AL(it isreadily seen that S and I commute, so since S2 = I2 = id,then also H2 = id, the identical transformation). The ele-ments of the configuration interchanged byH are summa-rized in Table I.

    Let O1 and O2 be the centres of circles 1 and 2. Sincethe circles 1 and 2 are determined by their construc-tion (in a unique way), they are interchanged byH, there-fore the rays AO1 and AO2 are symmetrical with respect

  • 8/3/2019 Sols2011D1

    3/3

    3

    to b. Denote by1 and 2 the radii of1 and 2. SinceO1AB = O2AC, we have 1/2 = AO1/AO2. On theother hand, from the definition ofP we have O1P/O2P =1/2 = AO1/AO2; this means that AP is the angle bisectorofO1AO2 and therefore ofB AC.

    The limiting, degenerated, cases are when the parallelline passes through A when P coincides with A; respec-

    tively when the parallel line is BC when P coincides withthe foot A BC of the angle bisector ofB AC (or anyother point on BC). By continuity, any point P on the opensegment A A is obtained for some position of the parallel,therefore the locus is the open segment A A of the angle bi-sector bofB AC.

    point K point Lline K L circlerayAB rayACpoint B point Epoint C point D

    segment B D segment ECarc B K segment E Larc C L segment DK

    TABLE I: Elements interchanged byH.

    A

    B C

    D EK L

    O1

    O2P

    b